Difference between revisions of "2021 Fall AMC 10B Problems/Problem 19"
Ihatemath123 (talk | contribs) |
Ihatemath123 (talk | contribs) m |
||
Line 10: | Line 10: | ||
Behind all this complex notation, all that we're really saying is that the first digit of something like <math>\sqrt[3]{123456789}</math> has the same first digit as <math>\sqrt[3]{123456.789}</math> and <math>\sqrt[3]{123.456789}</math>. | Behind all this complex notation, all that we're really saying is that the first digit of something like <math>\sqrt[3]{123456789}</math> has the same first digit as <math>\sqrt[3]{123456.789}</math> and <math>\sqrt[3]{123.456789}</math>. | ||
− | The problem asks for < | + | The problem asks for |
+ | <cmath>B(\sqrt[2]{x}) + B(\sqrt[3]{x}) + B(\sqrt[4]{x}) + B(\sqrt[5]{x}) + B(\sqrt[6]{x}).</cmath> | ||
+ | |||
+ | From our previous observation, we know that | ||
<cmath>B(\sqrt[2]{x}) = B(\sqrt[2]{\frac{x}{100}} = B(\sqrt[2]{\frac{x}{10,000}} = B(\sqrt[2]{\frac{x}{1,000,000}} = \ldots .</cmath> | <cmath>B(\sqrt[2]{x}) = B(\sqrt[2]{\frac{x}{100}} = B(\sqrt[2]{\frac{x}{10,000}} = B(\sqrt[2]{\frac{x}{1,000,000}} = \ldots .</cmath> | ||
Therefore, <math>B(\sqrt[2]{x}) = B(\sqrt[2]{7.777 \dots})</math>. We can evaluate <math>B(\sqrt[2]{7.777 \dots})</math>, the leading digit of <math>\sqrt[2]{7.777 \dots}</math>, to be <math>2</math>. Therefore, <math>f(2) = 2</math>. | Therefore, <math>B(\sqrt[2]{x}) = B(\sqrt[2]{7.777 \dots})</math>. We can evaluate <math>B(\sqrt[2]{7.777 \dots})</math>, the leading digit of <math>\sqrt[2]{7.777 \dots}</math>, to be <math>2</math>. Therefore, <math>f(2) = 2</math>. | ||
Line 18: | Line 21: | ||
Therefore, <math>B(\sqrt[3]{x}) = B(\sqrt[3]{7.777 \ldots})</math>. We know <math>B(\sqrt[3]{7.777 \ldots}) = 1</math>, so <math>f(3) = 1</math>. | Therefore, <math>B(\sqrt[3]{x}) = B(\sqrt[3]{7.777 \ldots})</math>. We know <math>B(\sqrt[3]{7.777 \ldots}) = 1</math>, so <math>f(3) = 1</math>. | ||
− | + | Next, | |
+ | <cmath>B(\sqrt[4]{x}) = B(\sqrt[4]{7.777 \ldots})</cmath> | ||
+ | and <math>B(\sqrt[4]{7.777 \ldots}) = 1</math>, so <math>f(4) = 1</math>. | ||
+ | |||
+ | We also have | ||
+ | <cmath>B(\sqrt[5]{x}) = B(\sqrt[5]{777.777 \ldots})</cmath> | ||
+ | and <math>B(\sqrt[5]{777.777 \ldots}) = 3</math>, so <math>f(5) = 3</math>. | ||
+ | |||
+ | Finally, | ||
+ | <cmath>B(\sqrt[6]{x}) = B(\sqrt[6]{7.777 \ldots})</cmath> | ||
+ | and <math>B(\sqrt[4]{7.777 \ldots}) = 1</math>, so <math>f(6) = 1</math>. | ||
We have that <math>f(2)+f(3)+f(4)+f(5)+f(6) = 2+1+1+3+1 = \boxed{\textbf{(A) } 8}</math>. | We have that <math>f(2)+f(3)+f(4)+f(5)+f(6) = 2+1+1+3+1 = \boxed{\textbf{(A) } 8}</math>. |
Revision as of 10:22, 23 November 2021
Problem
Let be the positive integer , a -digit number where each digit is a . Let be the leading digit of the th root of . What is
Solution
For notation purposes, let be the number with digits, and let be the leading digit of . As an example, , because , and the first digit of that is .
Notice that for all numbers ; this is because , and dividing by does not affect the leading digit of a number. Similarly, In general, for positive integers and real numbers , it is true that Behind all this complex notation, all that we're really saying is that the first digit of something like has the same first digit as and .
The problem asks for
From our previous observation, we know that Therefore, . We can evaluate , the leading digit of , to be . Therefore, .
Similarly, we have Therefore, . We know , so .
Next, and , so .
We also have and , so .
Finally, and , so .
We have that .
~ihatemath123
See Also
2021 Fall AMC 10B (Problems • Answer Key • Resources) | ||
Preceded by Problem 18 |
Followed by Problem 20 | |
1 • 2 • 3 • 4 • 5 • 6 • 7 • 8 • 9 • 10 • 11 • 12 • 13 • 14 • 15 • 16 • 17 • 18 • 19 • 20 • 21 • 22 • 23 • 24 • 25 | ||
All AMC 10 Problems and Solutions |
The problems on this page are copyrighted by the Mathematical Association of America's American Mathematics Competitions.